Answered step by step
Verified Expert Solution
Link Copied!

Question

1 Approved Answer

refer to the files below (20 points) The market inverse demand curve is P(y) = 10 - 2y, and a monopolist's cost curve is y2

image text in transcribedimage text in transcribedimage text in transcribedimage text in transcribedimage text in transcribedimage text in transcribed

refer to the files below

image text in transcribedimage text in transcribedimage text in transcribedimage text in transcribedimage text in transcribedimage text in transcribed
(20 points) The market inverse demand curve is P(y) = 10 - 2y, and a monopolist's cost curve is y2 + 2. (a) What output level y maximizes the monopolist's revenue? What output level y maximizes the monopolist's profit? Identify which of the two output levels is lower, and explain why using economic intuition. (b) Suppose a second firm with cost curve y + 2 is considering entering the market. If after entry, the firms would compete a la Cournot, what would be the Cournot- Nash equilibrium output levels y1 and y2 of firms 1 and 2? What would be the equilibrium profits for each firm? Will firm 2 choose to enter the market? (c) Suppose that if firm 2 enters, both firms collude, choosing output levels that maxi- mize total profits and then split the profits equally between them. What would be the profits to each firm? Will firm 2 choose to enter the market in this case?(20 points) The market inverse demand curve is P(y) = 10 - 2y, and a monopolist's cost curve is y2 + 2. (a) What output level y maximizes the monopolist's revenue? What output level y maximizes the monopolist's profit? Identify which of the two output levels is lower, and explain why using economic intuition. (b) Suppose a second firm with cost curve y + 2 is considering entering the market. If after entry, the firms would compete a la Cournot, what would be the Cournot- Nash equilibrium output levels y1 and y2 of firms 1 and 2? What would be the equilibrium profits for each firm? Will firm 2 choose to enter the market? (c) Suppose that if firm 2 enters, both firms collude, choosing output levels that maxi- mize total profits and then split the profits equally between them. What would be the profits to each firm? Will firm 2 choose to enter the market in this case?70) When compared to a perfectly competitive market, a single-price monopoly with the same costs 70) produces output and charges - price. 4) a smaller, a lower B) a larger, a lower () a smaller, a higher D) a smaller, the same E) the same, a higher Price and cost (dollars per hamburger) 5.00 4.50 3.50 3/00- 2.50 2.00 1.50 1.00 . 0.50. Quantity (hamburgers per hour) 71) Suppose the Busy Bee Cafe is the monopoly producer of hamburgers in Huge, Oklahoma. The 71) above figure represents the demand, marginal revenue, and marginal cost curves for this establishment. What quantity will the Busy Bee produce to maximize its profit? 4) 20 hamburgers per hour B) 50 hamburgers per hour () 10 hamburgers per hour D) d hamburgers per hour. E) 30 hamburgers per hour 16 Price(dollars) Quantity units! 72) The above table gives the demand schedule for a monopoly. The demand is elastic at all prices 72) between A)5 and $1. B) $5 and $1. () $4 and $3. D) $6 and $1. E) $6 and $4. 73) The above table gives the demand schedule for a monopoly. The demand is inelastic over the entice price range between A) $6 and $4. B) $6 and $1. () $3 and $1. D) $4 and $3. E) $5 and $1.64) A single-price monopoly transfers 64) A) economic profit to the government. B) consumer surplus to producers. C) producer surplus to consumers. D) economic profit to deadweight loss. E) economic profit to consumers. 14 Price and cost (dollars per unit) 50.00 40.00 10.00 MR D 0 100 200 300 400 500 Quantity (units per hour) 65) The figure above shows the demand curve, marginal revenue curve, and marginal cost curve. 65) The amount of consumer surplus when the market has a monopoly producer is A) ace. B) bcef. C) bed. D) abf. E) acd. 66) The figure above shows the demand curve, marginal revenue curve, and marginal cost curve. 66) The amount of consumer surplus when the market has a monopoly producer is _ and the amount of consumer surplus when the market is perfectly competitive is A) abf; ace B) ace; bed C) ace; abf D) abf; bod E) bed; ace 67) Compared to a perfectly competitive market, a single-price monopoly sets 67) A) a higher price. B) a lower price. C) the same price. D) a price that might be higher, lower, or the same depending on whether the monopoly's marginal revenue curve lies above, below, or on its demand curve. E) a price that might be higher, lower, or the same depending on whether the monopoly's marginal cost curve lies above, below, or on its marginal revenue curve. 68) Compared to a perfectly competitive industry, a single-price monopoly produces 68) A) the same output. B) more output. C) less output. D) some amount that might be more, less, or the same depending on whether the monopoly's marginal revenue curve lies above, below, or on its demand curve. E) some amount that might be more, less, or the same depending on whether the monopoly's marginal cost curve lies above, below, or on its marginal revenue curve. 15 69) Mark owns a cattle ranch near Hugo, Oklahoma. Mark is currently producing beef at an output 69) level where marginal revenue exceeds marginal cost. In order to maximize his profit, Mark should A) decrease his output. B) shut down his ranch. C) increase his output. D) not change his output. E)52) The above figure represents the market for cable television in Oakland, Florida. Time Warner 52) Communications (TWC) is the sole provider of cable television to the residents of this Central Florida community. If TWC is left unregulated, how many households in Oakland are served? A) 40,000 B) 50,000 C) 10,000 D) 30,000 E) 20,000 53) If a regulatory agency sets the price equal to marginal cost for a natural monopoly, the 53) A) price is the same as the unregulated monopoly price. B) firm earns an economic profit, though not the maximum economic profit. C) firm earns a normal profit. D) firm earns the maximum economic profit. E) government might have to provide a subsidy to the firm to keep it in business. 54) Capture theory is 54) A) a model about perfect competition. B) a theory that explains behavior of competitive firms. C) an economic theory of regulation. D) the theory that regulators capture firms' attention by dictating a very low price. E) the same as the public interest theory. 55) A monopoly creates a deadweight loss because the monopoly 55) A) sets a price that is too low. B) produces less than the efficient quantity. C) produces more than the efficient quantity. D) does not maximize profit. E) earns a normal profit. 12 56) Which of the following explains why the marginal cost pricing rule results in an economic loss 56) for a natural monopoly? A) The ATC curve is downward sloping throughout the relevant range, therefore the MC is lower than the ATC B) The MC is constant and equal to price. C) Because output is determined by settingMC equal to the price, consumer surplus is maximized. D) The demand curve is downward sloping, therefore price falls as quantity increases. E) The firm'sMR is always less than its price. 57) natural monopolies is a commonly used, potential solution to the problems presented 57) by natural monopolies. A) Giving incentives to firms to become B) Breaking up firms that are C) Regulating D) Refusing to grant patents to E) Outlawing price discrimination by 58) With perfect price discrimination, the level of output 58) A) is the same as the amount produced by any monopoly that price discriminates. B) equals the amount produced by a single-price monopoly. C) is the same as the amount produced in a perfectly competitive market. D) exceeds the efficient quantity. E) is unknown. 59) Comparing a perfectly competitive market to a single-price monopoly with the same costs, we 59) see that A) the monopoly market always is more efficient in the use of resources. B) the monopoly market achieves efficiency in resource use while perfectly competitive market does not. C) both markets are equally efficient in their use of resources. D) the perfectly competitive market achieves efficiency in resource use while the monopoly market does not. E) None of the above answers is correct because comparing a perfectly competitive market to a monopoly is impossible.44) With an average cost pricing rule, the quantity produced by the natural monopoly is 44) the quantity produced with a marginal cost pricing rule. A) less than B) greater than C) not comparable to D) equal to E) greater than in the long run and less than in the short run than 45) Because of the number of firms in monopolistic competition 45) A) each firm has a large market share. B) it is possible for the firms to collude. C) one firm has the ability to dictate market conditions. D) each firm must carefully monitor what its competitors do. E) no one firm can dominate the market. 46) If a large number of firms are competing, the market could be 46) A) monopolistic competition or monopoly. B) perfect competition or monopoly. C) oligopoly or monopoly. D) perfect competition or monopolistic competition. E) monopolistic competition or oligopoly. 47) With a natural monopoly 47) A) no regulation is necessary because it is a natural monopoly. B) regulation takes the form of breaking the company into several competing firms. C) regulation takes the form of forcing the company out of business. D) regulation can take the form of average cost pricing to allow coverage of costs. E) regulation takes the form of forcing competition from new firms. 48) If a natural monopoly is regulated using 48) A) a total cost pricing rule, the firm will exit the industry. B) a marginal cost pricing rule, the firm maximizes its profit. C) an average cost pricing rule, the firm maximizes its profit. D) a marginal cost pricing rule, the firm incurs an economic loss. E) an average cost pricing rule, the firm incurs an economic loss. 49) Price cap regulation is defined as regulation that 49) A) imposes a price ceiling on the regulated firm. B) is essentially the same as rate of return regulation. C) uses average cost pricing to ensure costs are covered. D) uses marginal cost pricing to ensure efficient output. E) encourages firms to exaggerate costs to increase profits. 50) The process of price cap regulation includes which of the following? 50) i. a price ceiling ii. marginal cost pricing. iii. average cost pricing A) i and ii B) i and iii C) ii only D) ii and iii E) i only 11 Price and cost (dollars) 50 40 30 20 ATC 10 MC MR D 10 20 30 40 50 Quantity (thousands of households) 51) The above figure represents the market for cable television in Oakland, Florida. Time Warner 51) Communications (TWC) is the sole provider of cable television to the residents of this Central Florida community. If TWC is left unregulated, what is the price of cable television in Oakland? A) $10 B) $20 C) $50 D) $30 E) $4030) The figure above shows the market demand curve and the ATC curve for a firm. If all firms in 30) the market have the same ATC curve, the efficient scale for one firm is units per hour. A) 2,000 B) 4,000 C) 8,000 D) 10,000 E) more than 10,000 31) Which of the following is correct? 31) A) In the long run, a firm in monopolistic competition earns zero economic profit and its price is equal to the minimum average total cost. B) In the long run, a firm in monopolistic competition can earn an economic profit because of product differentiation. C) A firm in perfect competition operates at maximum average total cost in the long run. D) In the long run, a firm in monopolistic competition maximizes its profit at a point where price is equal to average total cost but the average total cost is not minimized. E) A firm in monopolistic competition does not have excess capacity in the long run. 8 32) A cartel is most likely to occur in 32) A) perfect competition as firms compete by reducing cost. B) oligopoly as firms compete to lower price and increase their own profits. C) monopolistic competition where firms collude to increase profits. D) monopoly because it faces no competition. E) oligopoly as firms act together to raise prices and increase profits. 33) When firms in monopolistic competition are making an economic profit, firms will 33) A) enter the industry, and demand will decrease for the original firms. B) exit the industry, and demand will increase for the firms that remain. C enter the industry and then will exit the industry. D) enter the industry, and demand will increase for the original firms. E) exit the industry, and demand will decrease for the firms that remain. 34) Herb's Inc. has a large share of its market and is tempted to collude with the few firms that are in 34) its market. Herb's operates in A) a perfectly competitive market. B) collusively protected market. C) a monopoly market. D) a monopolistically competitive market. E) an oligopoly. 35) A cartel is 35) A) a market structure with a large number of small firms. B) a market with only two firms. C) a group of firms acting together to raise price, decrease output, and increase economic profit. D) a market structure with a small number of large firms. E) another name for an oligopoly

Step by Step Solution

There are 3 Steps involved in it

Step: 1

blur-text-image

Get Instant Access to Expert-Tailored Solutions

See step-by-step solutions with expert insights and AI powered tools for academic success

Step: 2

blur-text-image

Step: 3

blur-text-image

Ace Your Homework with AI

Get the answers you need in no time with our AI-driven, step-by-step assistance

Get Started

Recommended Textbook for

Microeconomics Principles, Problems and Policies

Authors: Campbell R. McConnell, Stanley L. Brue, Sean Masaki Flynn

20th edition

978-0077660819, 77660811, 978-1259450242

More Books

Students also viewed these Economics questions

Question

Evaluate A(x), where x2 A(x) = -x2 t cos (t) dt.

Answered: 1 week ago